Practice Board Questions

Cardiology

Which of the following therapies has been demonstrated to improve survival and hospital discharge with favorable neurologic outcomes in out of hospital cardiac arrest?

A. Amiodarone
B. Epinephrine
C. Hypothermia
D. Time to initial defibrillation < 10 min
E. Vasopressin

Answer Explained

Hypothermia has been confirmed as a benefit following out of hospital arrest in 2 studies. These trials showed that rapidly cooling to 32-34⁰C and maintaining these temperatures for 12-24 hours decreased in-hospital mortality and patients were 40-85% more likely to have good neurologic outcomes upon hospital discharge. None of these medications have ever demonstrated any effects on neurologic outcomes and only defibrillation within 5 minutes has the greatest likelihood for good neurologic outcomes.

A 19 yo woman presents with complaints of DOE and mild fatigue. She has no significant medical history, does not use tobacco and takes no regular medications. On exam, her lungs are clear and cardiac exam reveals a II/VI systolic murmur at the 2nd left intercostal space, which varies with inspiration. What is the most likely diagnosis for this patient?

A. Atrial septal defect
B. Pulmonary stenosis
C. Bicuspid aortic valve
D. Ventricular septal defect
E. Hypertrophic obstructive cardiomyopathy

Answer Explained

This patient presents with a systolic murmur that varies with respiration. This makes it likely that the etiology is right sided, and given the location, pulmonary stenosis is more likely than tricuspid regurgitation. These right sided murmurs vary with respiration because filling of the right heart is influenced by changes in thoracic pressure.

A 31-year-old man presents with repetitive generalized motor convulsions that continue for 35 minutes until 2 mg of lorazepam are administered intravenously. Which of the following should be administered next??

A. Phenytoin intravenously
B. Carbamazepine orally
C. Pentobarbital intravenously
D. Ethosuximide orally
E. Diazepam rectally

Answer Explained

Administration if IV lorazepam should be followed by the administration of phenytoin (or fosphenytoin) to control status epilepticus because the duration of action of lorazepam is limited. Carbamazepine is an effective anticonvulsant, but it cannot be given IV or IM. IV pentobarbital can be used but because the patient is not currently convulsing, induction of barbiturate coma is not indicated. Ethosuximide is indicated for the treatment of absence but not generalized tonic-clonic seizures. Rectal diazepam is used to abort seizures temporarily, especially in children.

A 58 year-old woman presents with a 5-month history of progressive difficulty walking. She denies pain, although she does note mild paresthesias in her feet. She has no bowel or bladder symptoms. On questioning, she thinks her hands may not be as strong as they should be. On examination, strength in her hands and distal legs is 4/5. There is mild sensory loss to all modalities to her knees. Muscle stretch reflexes are absent at the knees and ankles, and there is no Babinski response. Which of the following diagnoses is most likely?

A. Guillain-Barre syndrome
B. Chronic inflammatory demyelinating polyneuropathy
C. Vitamin B12 deficiency
D. Polymyositis
E. Cervical myelopathy

Answer Explained

Chronic inflammatory demyelinating polyneuropathy is a cause of progressive predominantly motor neuropathy leading to distal weakness with associated loss of reflexes and possible sensory loss. Typically, bowel and bladder control is retained, and there is a paucity of pain. Vitamin B12 deficiency causes loss of lower extremity position and vibration perception. There can be mild leg weakness with evidence of an upper motor neuron lesion such as an extensor plantar response combined with hyporeflexia associated with a concurrent peripheral neuropathy. Cervical mylopathy causes lower extremity weakness with hyperreflexia. Sensory loss and bowel and bladder disordered function can be present. There is often neck discomfort. Upper extremity symptoms and signs can be present. Polymoysitis causes predominantly proximal weakness with sensory changes. Myalgias can be present. By definition, the weakness associated with Guillain-Barre syndrome does not progress beyond approximately 4 weeks. Therefore, the chronicity of this patient’s progressive symptoms excludes this diagnosis.

A 38-year old man with depression presents to the ER with flushing, diarrhea, sweating and muscle rigidity. During the physical exam, he says that he began seeing a new psychiatric PA because his sertraline was not working for him. His PA gave him a different medication, but the patient decided to use both medicines to get rid of the depression . Which of the following medications did his PA most likely prescribe for this patient?

A. Lithium
B. Citalopram
C. Nortriptyline
D. Tranylcypromine
E. Trazodone

Answer Explained

Tranylcypromine is a nonselective MAOI and may lead to serotonin syndrome if take with SSRIs due to an overall increase in serotonin. Serotonin syndrome is characterized by mental status changes, autonomic changes (fever, diaphoresis, tachycardia), and neuromuscular changes (tremor or rigidity).

Incorrect answers include:

Lithium – associated with tremor, hypothyroidism and nephrogenic diabetes insipidus.

Citalopram – SSRI associated with anxiety, insomnia, tremor, and nausea.

Nortriptyline – a tricyclic antidepressant associated with the 3 C’s: convulsions, coma, and cardiotoxicity.

Trazodone – a heterocyclic associated with sedation, nausea, priapism, and postural hypotension.

CME Webinar #8 – Gout Update

New treatment guidelines for gout recommend decrease uric acid levels to less than?

A. 4.0 mg/dL
B. 6.0 mg/dL
C. 8.0 mg/dL
D. 10.0 mg/dL
E. 12.0 mg/dL

Answer Explained

New treatment guidelines for gout recommend decreasing uric acid levels to less than 6.0 mg/dL .

All the following are gout risk factors EXCEPT?

A. Thiazide diuretics
B. Niacin
C. Dairy products
D. A&B
E. All the above

Answer Explained

Risk factors for gout include (A&B) thiazide diurectics, niacin and seafood. Protective factors for gout include: coffee, dairy products and low BMI.

Which of the following is the FDA approved colchicine dosage for an acute gout flare up?

A. One tablet at onset and daily x 3 days
B. One tablet bid x 3 days
C. Two tablets bid x 3 days
D. Two tablets at onset and one tablet one hour later
E. Two tablets at onset and daily x 3 days

Answer Explained

FDA indications for colchicine include taking two tablets at onset and one tablet one hour later. The maximum dose in one hour is 1.8mg.

All of the following can be used to describe gout crystals EXCEPT?

A. Negatively birefringent on polarized microscopy
B. Composed of calcium pyrophoshate
C. Shaped liked needles
D. All the above
E. None of the above

Answer Explained

Migraine and Cluster Headaches (A&B) are indications for sumitriptan. Basilar and Hemiplegic headaches are contraindications.

CME Webinar #7 – Migraine Headache

Which of the following is Sumitriptan indicated for?

A. Migraine Headache
B. Cluster Headache
C. Basilar & Hemiplegic Migraine
D. A&B
E. All the above

Answer Explained

Migraine and Cluster Headaches (A&B) are indications for sumitriptan. Basilar and Hemiplegic headaches are contraindications.

CME Webinar #7 – Migraine Headache

Which of the following is Sumitriptan indicated for?

A. Migraine Headache
B. Cluster Headache
C. Basilar & Hemiplegic Migraine
D. A&B
E. All the above

Answer Explained

Migraine and Cluster Headaches (A&B) are indications for sumitriptan. Basilar and Hemiplegic headaches are contraindications.

What is the #1 type of diagnosed primary brain tumor?

A. Meningioma
B. Glioma
C. Pituitary adenoma
D. Malignant melanoma
E. Small cell carcinoma

Answer Explained

The number one type of diagnosed primary brain tumor is Glioma making up 54-60% of tumors. Gliomas can be either benign or malignant and include: ependymomas, astrocytomas including glioblastoma multiforme, oligodendrogliomas and mixed gliomas .

What is the recommended maximum daily dose of sumitriptan (oral / intranasal / subQ)?

A. 100mg / 20mg / 6mg
B. 100mg / 40mg / 12mg
C. 200mg / 20mg / 6mg
D. 200mg / 40mg / 12mg
E. 400mg / 80mg / 18mg

Answer Explained

The recommended maximum daily dose for oral sumitriptan is 200mg.
The recommended maximum daily dose for nasal sumitriptan is 40mg.
The recommended maximum daily dose for subQ sumitriptan is 12mg.

Increased Deep Tendon Reflexes can occur with?

A. Alcoholism
B. Upper Motor Lesions
C. Diabetes Mellitus
D. Vitamin Deficiencies
E. Lead poisioning

Answer Explained

Of the choices, only Upper Motor Lesions can increase Deep Tendon Reflexes. Deep Tendon Reflexes are decreases with: peripheral neuropathy (most common cause), diabetes mellitus, alcoholism, amyloidoisis, uremia, vitamin deficiencies (pellagra, beriberi), pernicious anemia, and toxins (lead, arsenic, isoniazid).

Known vasodilating triggers of a migraine headache include?

A. Alcohol
B. Nitroglycerine
C. Ergotamine
D. A&B
E. All the above

Answer Explained

Vasodilating triggers of migraines include Alcohol and Nitroglycerine (A&B). Ergotamine is a known vasoconstrictor.

Intracranial hypertension is diagnostic when CSF opening pressure is?

A. > 20 mm H2O
B. > 50 mm H2O
C. > 100 mm H2O
D. > 250 mm H2O
E. > 400 mm H2O

Answer Explained

In adults and children over 8, normal opening CSF pressure is < 200. Obese patients may have pressures up to 250 mm H20. Intracranial hypertension is diagnostic when the CSF opening pressure is > 250 mm H2O.

Contraindications for sumitriptan include?

A. Ischemic heart disease
B. TIA
C. Uncontrolled hypertension
D. A&B
E. All the above

Answer Explained

Contraindications for sumitriptan include Ischemic heart disease, Cerebrovascular syndromes (strokes, TIA), and Uncontrolled hypertension (All the above). Other contraindications include: peripheral vascular disease, severe hepatic impairment, use within 24 hours of ergotamine derivaties, management of hemiplegic or basilar migraine, and concurrent administration or within 2 weeks of discontinuing an MAO type A inhibitor.

The majority of meningiomas are ?

A. Benign
B. Asymptomatic
C. Schwanomas
D. A&B
E. All the above

Answer Explained

Most meningiomas are Benign (92%) and Asymptomatic (A&B). Schwanomas are a type of nerve sheath tumor and NOT a type of meningioma.

Combination of sumitriptan and which of the following medications should be avoided (Risk X)?

A. Ergot derivatives
B. MAO Inhibitors
C. Metoclopromide
D. A&B
E. All the above

Answer Explained

Drug interactions with sumitriptan that give a Risk X and should be AVOIDED indclude Ergot derivaties and MAO Inhibitors(A&B). Drug interactions with sumitriptan that give a Risk C and therapy should be monitored include: antipsychotics, metoclopramide, and serotonin modulators.

Bitemporal hemianopia commonly occurs with which type of tumor (if large enough)?

A. Glioma
B. Pituitary adenoma
C. Meningioma
D. Neurofibroma
E. All the above

Answer Explained

Bitemporal hemianopia is a symptom the can occur with Pituitary adenmas, if they are large enough to push on the optic chiasm.

CME Webinar #4 – Immunizations

Which of the following immunizations should be avoided if the patient has an egg allergy?

A. Varicella
B. Measles, Mumps, Rubella
C. Haemophilus influenzae
D. Influenza
E. Meningococcal

Answer Explained

Influenza vaccination should NOT be given if patients have an egg allergy.  Measles, Mumps, & Rubella (MMR), varicalla, and polio vaccines should be avoided if patient has a neomycin allergy.

All of the following immunizations can be given at the 2, 4, and 6 month age EXCEPT?

A. DTaP
B. Haemophilus influenzae
C. Measles, Mumps, Rubella
D. Polio
E. All the above can be given at 2, 4, and 6 months

Answer Explained

Measles, Mumps, & Rubella (MMR) vaccine should NOT be given at the 2, 4, and 6 months but rather at one year (12-15 months) and repeated at 4-6 years.

The following vaccines can be given at 2, 4, and 6 months:

  • DTAP: 5 doses with last 2 at 15-18 months and 4-6 years
  • Haemophilus influenzae B: 4 doses with last dose at 12-15 months
  • Pneumococcal: 4 doses with last dose at 12-15 months
  • Polio: 4 doses with last dose at 4-6 years
  • Rotavirus: 2 or 3 doses at 2, 4, and/or 6 months

In infants and children, what is the maximum daily dose of paracetamol that can be given in 24 hours?

A. 1 mg
B. 2.6 mg
C. 4 mg
D. 1 gram
E. 2.6 grams

Answer Explained

Acetaminophen is more commonly known around the world at paracetamol.  In adults, the maximum daily dose of acetaminophen/paracetamol is 4 grams, usually divided 325 or 650 mg PO/PR every 4-6 hours PRN.  In infants and children, the maximum daily dose of acetaminophen/paracetamol is 2.6 grams, usually divided 10-15 mg/kg/dose every 4-6 hours PRN. 

In children, the CDC recommends starting chelation therapy if the lead level is at least?

A. 10 μg/dL
B. 25 μg/dL
C. 45 μg/dL
D. 65 μg/dL
E. 100 μg/dL

Answer Explained

In May of 2012, the CDC changed their definition of a “blood lead level of concern” in children from 10 μg/dL to 5 μg/dL.  What did not change was their recommendation of chelation therapy that should be started when the child’s lead level is 45 μg/dL or higher.

Which of the following is the recommended treatment for pertussis in a 3-week old child?

A. Erythromycin
B. Azithromycin
C. Clarithromycin
D. Bactrim
E. Vancomycin

Answer Explained

The only medication recommended to treat pertussis in children less than 1 month is Azithromycin at a dose of 10mg/kg/day.   Clarithromycin and Erythromycin are appropriate treatments but are not recommended for children under one month.  Bactrim is contraindicated for children less than 2 months.  Vancomycin is not an appropriate treatment for pertussis.

Webinar #3 – Heparin Induced Thrombocytopenia

Webinar #2 – Infective Endocarditis

Which of the following antibiotics is recommended for the treatment of endocarditis by HACEK organisms?

A. Ceftriaxone (Rocephin)
B. Vancomycin (Vancocin)
C. Ampicillin-sulbactam (Unasyn)
D. A&B
E. All the above

Answer Explained

Which of the following is the normal range for a Vancomycin trough?

A.
B.
C.
D.
E.

Answer Explained

Which of the following is NOT one of the four staining steps of a gram stain?

A. Crystal Violet
B. Iodine
C. Alcohol
D. Safranin
E. Crimson

Answer Explained

All of the following are possible adverse reactions with Gentamicin EXCEPT?

A. Hepatotoxicity
B. Nephrotoxicity
C. Neurotoxicity
D. Suprainfection
E. All the above are adverse reactions with Gentamicin

Answer Explained

Possible adverse reactions with Gentamicin include: pain at the injection site and thromophlebitis (~1%). Less common adverse reactions include Nephrotoxicity, Neurotoxicity and Suprainfections of other bacteria or fungi with prolonged use. Hepatotoxicity is not an adverse reaction with Gentamicin.

What are painful red-purple immunologic lesions on the hands or feet of patients with endocarditis?

A. Janeway lesions
B. Osler nodes
C. Splinter hemorrhages
D. Roth spots
E. None of the above

Answer Explained

Webinar #1 – Acid/Base Disorders

A 78 year-old man has been diagnosed with localized prostate cancer. He is not a surgical candidate, and he is refusing radiation therapy. Which of the following would prostate cancer respond best to?

A. Use of saw palmetto
B. Radioimmunotherapy with ibritumomab tiuxetan
C. Treatment with finasteride
D. Tamoxifen
E. Androgen blockade

Answer Explained

Prostate cancer cells are responsive to testosterone withdrawal. Total androgen blockade can be accomplished by the administration of luteinizing hormone-releasing hormone antagonists and drugs that would block the biosynthetic pathway of testosterone production. Tamoxifen is an antiestrogen agent that does not have a role in the treatment of prostate cancer. Both saw palmetto and finasteride are used for BPH and do not have significant effects on prostate cancer cells.

A 56 year-old woman has an elevated serum calcium level of 12.2 mg/dL. She has no history of any ilnness or treatment associated with hypercalcemia. Which of the following studies would be most helpful in making a diagnosis of primary hyperparathyroidism?

A. 24 hr urine calcium excretion
B. Serum ionized calcium
C. Serum intact parathyroid hormone (PTH)
D. Computed tomography of the neck
E. Serum phosphate

Answer Explained

An elevated level of serum intact PTH, in the absence of renal failure or other cause of secondary hyperparathyroidism, is strong evidence for primary hyperparathyroidism. A high normal PTH is also compatible with primary hyperparathyroidism because the PTH should be suppressed due to the hypercalcemia and is, there fore, inappropriately elevated. In hypercalcemia of other causes (e.g., with cancer, sarcoidosis, or excessive vitamin D intake), the PTH level is suppressed by the hypercalcemia nd is low (or normal).

 

Antiarrhythmic drugs can be separated into four main groups, types I to IV. All of the following are type I antiarrhythmic drugs EXCEPT:

A. Quinidine
B. Lidocaine
C. Procainamide
D. Phenytoin
E. Propranolol

Answer Explained

Propranolol is a beta blocker and beta blockers form the antiarrhythmic group II.

A 71 year old male moves to your state and transfers as a new patient. He has been successfully treated for exercise-induced angina for the past 6 years. About 3 days ago he notes being woken up in the middle of night with chest pain. Which of the following medications would be useful in preventing this patient's nocturnal angina?

A. Amyl nitrate
B. Esmolol
C. Nitroglycerin (sublingual)
D. Nitroglycerin (transdermal)
E. Hydralazine

Answer Explained

Transdermal nitroglycerin can sustain blood levels for as long as 24 hours but because tolerance can occur, it is recommended that the patch be removed after 10 to 12 hours to allow recovery of sensitivity. Esmolol (IV), amyl nitrite, and sublingual nitroglycerin all have short durations of actions. Hydralazine may actually precipitate an anginal attack.

A 68 year old man recently had a myocardial infarction. Which of the following medications would be the most appropriate prophylactic antiarrhythmic therapy?

A. Procainamide
B. Lidocaine
C. Metoprolol
D. Verapamil
E. Quinidine

Answer Explained

Numerous studies show that Beta blockers such as Metoprolol help prevent cardiac arrhythmias that occur related to myocardial infarctions. None of the other medications listed have been proven to be more effective.

Which one of the following antihypertensive drugs can precipitate a hypertensive crisis following abrupt cessation of therapy?

A. Clonidine
B. Enalapril
C. Diltiazem
D. Losartan
E. Hydrochlorothiazide

Answer Explained

Increased sympathetic nervous system activity occurs if clonidine is abruptly stopped after prolonged administration causing uncontrolled elevation in blood pressure. Patients should be slowly weaned from clonidine while other antihypertensive medications are initiated.

Patients with heart failure often have compensatory increases in heart rate and renin release. Which of the following medications can help prevent this?

A. Digoxin
B. Enalapril
C. Dopamine
D. Metoprolol
E. Dobutamine

Answer Explained

Heart failure usually causes a decrease in cardiac output and the body will naturally try to compensate by sympathetic stimulation. Beta blockers such as metoprolol prevent this increased heart rate and renin release. Enalapril is an ACE-inhibitor that will actually increase renin release. Digoxin slows the heart rate because of its vagomimetic effects, but does not affect renin release. Dopamine and Dobutamine are positive inotropic medications and neither decreases heart rate or affects renin release.

Which medication causes a decrease in liver triacyglycerol synthesis by limiting available free fatty acids needed as building blocks?

A. Fenofibrate
B. Niacin
C. Cholestyramine
D. Gemfibrozil
E. Atorvastatin

Answer Explained

The primary producer of circulating free fatty acids is lipolysis in adipose tissue. The liver normally utilized these free fatty acids as a major precursor for triacylglycerol synthesis. Niacin strongly inhibits adipose lipolysis and hence decreases the free fatty acids needed in liver triacylglycerol synthesis and VLDL production.

A 23 year old college male comes to your clinic because he and his friends are going mountain climbing in the Andes next week. He heard there is a medication you can take to help prevent acute mountain sickness. Which of the following would you prescribe?

A. Anticholinergic
B. Thiazide diuretic
C. Loop diuretic
D. Carbonic anhydrase inhibitor
E. Beta blocker

Answer Explained

Acetazolamine is a carbonic anhydrase inhibitor that is used prophylactically for severaly days before an ascent above 10,000 feet. This treatment helps prevent nausea and cerbral and pulmonary complications associated with acute mountain sickness.

Which of the following is NOT one of the seven metered-dose inhalers (MDIs) that the FDA has scheduled the removal of between June 1010 and December 2013?

A. Nedocromil
B. Triamcinolone
C. Cromolyn
D. Albuterol/ipratropium
E. Proair HFA

Answer Explained

Because older inhalers contain an ozone-depleating chlorofluorocarbon propellant they are being discontinued by the FDA. Proair HFA is the is the NEW formulary of albuterol sulfate that contains the environmentally friendly hydrofluoroalkanes.

Which of the following is/are long-acting B2-agonist(s)?

A. Salmeterol
B. Formoterol
C. Omalizumab
D. A&B
E. All the above

Answer Explained

A&B, both Salmeterol and Formoterol are long long-acting B2-agonists. Omalizumab is a recombinant DNA-derived monoclonal antibody that selectively binds to human immunoglobulin E (IgE), and may be used to treat moderate to severe allergic asthma in patiients who are poorly controlled with conventional therapy.

Which of the following adverse effects is specific to the biguanide diabetic drug metformin (Glucophage) therapy?

A. Hypoglycemia
B. Somulence
C. Nausea
D. Lactic acidosis
E. All the above

Answer Explained

Lactic Acidosis is the most dangerous adverse effect of metformin administration with death resulting in approximately 50 percent of individuals who develop lactic acidosis while on this drug. Metformin does not induce insulin production; thus, administration does not result in hypoglycemic events. Some nausea, vomiting, and diarrhea may develop but is usually not severe AND N/V/D is not specific to metformin. Metformin does not induce sleepiness.

Serious adverse effects of oral contraceptives include:

A. Thromboembolic complications
B. Stroke
C. Early or mid-cycle bleeding
D. A&B
E. All the above

Answer Explained

Serious adverse effects of oral contraceptives include A&B, thromboembolic complications (DVT), stroke, and myocardial infarction. These risks are increased in women who smoke. Early or mid-cycle bleeding are effects of estrogen deficiency.

Which of the following is the most likely side effect of first generation over-the-counter antihistamines in infants and children?

A. Reye's syndrome
B. Cholinergic effects
C. Paradoxical CNS stimulation
D. Nausea
E. Diarrhea

Answer Explained

Typically, first generation OTC antihistamines have a sedating effect because of passage into the CNS. However, in some individuals, especially infants and children, paradoxical CNS stimulation occurs and is manifested by excitement, euphoria, restlessness, and confusion. For this reason, use of first generation OTC antihistamines has declines, and second generation product usage has increased. Reye’s syndrome is a systemic response to a virus. First generation OTC antihistamines do not exhibit a cholinergic effects and do not commonly cause nausea or diarrhea.

Which of the following is the most serious adverse effect of tricyclic antidepressant (TCA) overdose?

A. Seizures
B. Hyperpyrexia
C. Metabolic acidosis
D. Cardiac arrhythmias
E. Pulmonary Fibrosis

Answer Explained

Excessive ingestion of TCAs result in life-threatening cardiac arrhythmias with wide QRS complex tachycardia. TCA overdose can induce seiures, but they are typically not life-threatening. TCAs do not cause an elevation in body temperature, metabolic acidosis or pulmonary fibrosis.

Which of the following best describes the hypomagnesemia in association to taking a prolonged proton pump inhibitor?

A. It is not associated
B. It is irreversible
C. It is always symptomatic
D. All the above
E. None of the above

Answer Explained

None of the above. Prolonged use of a proton pump inhibitor (PPI) can lead to hypomagnesemia as noted by the FDA. While most patients who take PPIs for a long time do not develop hypomagnesemia, about 30 cases of severe hypomagnesemia have been reported in long-term PPI users. When the PPI was stopped, serum magnesium levels returned to normal in less than 2 weeks. The exact mechanism is unknown. Patients also taking other drugs that cause hypomagnesemia, such as diuretics and digoxin, may be at increased risk.

Hypomagnesemia may be associated with?

A. Hypocalcemia
B. Hypokalemia
C. Weakness
D. All the above
E. None of the above

Answer Explained

All the above. Hypomagnesemia is often accompanied by hypocalcemia and hypokalemia. Patients with hypomagnesemia often do not have symptoms but they may include muscle weakness, tremor, muscle cramps, carpopedal spasm, tetany, seizures, and cardiac conduction disturbances and arrhythmias.

A 63 year-old, 73 kg white male just underwent a total left knee replacement and is taking IV acetaminophen 650 mg q 4hr and morphine as needed for post-operative pain. What is one risk of IV acetaminophen?

A. Hepatoxicity because the dosage of acetaminophen is higher than recommended
B. Hepatoxicity because morphine increases serum concentrations of acetaminophen
C. Acetaminophen may mask a post-operative fever that could be a sign of infection
D. Acetaminophen could have an additive effect on respiratory depression due to morphine
E. None of the above

Answer Explained

Acetaminophen may mask a post-operative fever that could be a sign of infection due to its antipyretic effects. IV acetaminophen does not affect respiratory depression and has not been associated with hepatotoxicity, but overdose can cause serious or fatal hepatic injury. IV analgesic adult dosage is 1000mg q6h or 650 mg q4hr (max: 4,000 mg/d). Given in conjunction with an opiod for moderate to severe pain, it has been shown to have an opiod-sparing effect.

During a workup for infertility, a 35 year-old man is noted to have a solid tumor in the anterior aspect of his left testis. What is the most likely diagnosis?

A. Testicular torsion
B. Cyst of the epididymis
C. Epididymo-orchitis
D. Lipoma of the cord
E. Testicular cancer

Answer Explained

Painless solid tumors in the testes are testicular caner until proven otherwise. Nonhematogenous testicular tumors are divided into 2 categories – germ cell tumors (seminoma, nonseminoma [embryonal, choriocarcinoma, teratoma, teratocarcinoma, yolk sac tumors]) and nongerm cell tumors (Leydig cell or Sertoli cell). There is no mass within the testis with torsion. Epididymitis presents with a painful tender testis.

A 44 year-old male comes to the clinic because his face is slightly swollen and he has some mild shortness of breath. He works in a factory and has never smoked. On examination, this vital signs are all normal and pupils are equal and reactive to light. Dilated venis are noted around the shoulders, upper chest, and face. An x-ray of the chest reveals an opacity in the superior mediastinum. What is the mot likely diagnosis?

A. Thymoma
B. Neurogenic tumor
C. Lymphoma
D. Teratodermoid tumor
E. Pheochromocytoma

Answer Explained

The most common cause of primary mediastinal tumor is a neurogenic tumor (20-25%), and 10% are malignant (more likely in children). They usually arise form an intercostal nerve or sympathetic ganglion. Varieties of neurogenic turmors include schwannoma, neurofibroma, ganglioneuroma, and neuroblastoma. Next in frequency (of primary mediastinal tumors) are thymoma, congenital cysts, and lymphoma.

At surgery for a right inguinal hernia, a 70 year-old man is found to have a hernia sac that in not independent of the bowel wall. The cecum forms part of the wall of the sac. Which of the following describes this hernia?

A. Incarcerated
B. Irreducible
C. Sliding
D. Interstitial
E. None of the above

Answer Explained

A sliding hernia refers to the peritoneum that slides along with the hernia in its passage along the cord. The viscus forms part of the wall of the sac. The peritoneum should not be removed from the bowel wall, because devascularization may occur.